Re: [obm-l] Teoria dos numeros

2020-05-22 Por tôpico Anderson Torres
Em ter., 19 de mai. de 2020 às 15:52, Israel Meireles Chrisostomo escreveu: > > Olá pessoal.Ultimamente tenho pensado em como provar que a tangente de um > arco racional diferente de zero é sempre irracional. Cê diz que se r é racional então tan(r) é irracional (exceto se r=0)? Acho que dá

[obm-l] Teoria dos numeros

2020-05-19 Por tôpico Israel Meireles Chrisostomo
Olá pessoal.Ultimamente tenho pensado em como provar que a tangente de um arco racional diferente de zero é sempre irracional.Eu consegui chegar no seguinte: Se r é real diferente de zero e s é inteiro diferente de zero, então ou tan(r-1/2s) ou tan(r) é irracional. Daí então eu tomo um r

Re: [obm-l] teoria dos numeros

2020-03-30 Por tôpico Carlos Victor
Basta fazer (2^3-1)^2n+(2^3+1)^2n -2 e usar binômio de Newton. Em 28/03/2020 13:55, Israel Meireles Chrisostomo escreveu: > Eu sei resolver o problema abaixo,porém não sei se é a forma mais simples de > se fazer.Vcs poderiam por favor colocar suas soluções nos comentários dessa >

Re: [obm-l] teoria dos numeros

2020-03-30 Por tôpico Pedro José
Boa noite! errata: Ao invés de: 128=2^7 então 2^7| 49^{n} + 81^{n} −2<==> x= 2^7| 49^{n} + 81^{n}=2 mod2^7 128=2^7 então 2^7| 49^{n} + 81^{n} −2<==> x= 49^{n} + 81^{n}=2 mod2^7 Saudações, PJMS Em dom., 29 de mar. de 2020 às 14:04, Pedro José escreveu: > Bom dia! > Prove que 128 divide 49^{n}

Re: [obm-l] teoria dos numeros

2020-03-29 Por tôpico Pedro José
Bom dia! Prove que 128 divide 49^{n} + 81^{n} −2, para todo n ≥ 1. 128=2^7 então 2^7| 49^{n} + 81^{n} −2<==> x= 2^7| 49^{n} + 81^{n}=2 mod2^7 x= a + b , a= 49^n e b=81^n a= (64-15)^n = n(-1)^n*n*64*(15)^(n-1) + (-1)^n*15^n mod2^7; pois, os demais termos do binômio de Newton terão o fator (2^6)^m

[obm-l] teoria dos numeros

2020-03-28 Por tôpico Israel Meireles Chrisostomo
Eu sei resolver o problema abaixo,porém não sei se é a forma mais simples de se fazer.Vcs poderiam por favor colocar suas soluções nos comentários dessa publicação? O problema é o seguinte: Prove que 128 divide 49^{n} + 81^{n} −2, para todo n ≥ 1.Se possível não use indução, pois eu já estou

Re: [obm-l] Teoria dos numeros

2018-03-28 Por tôpico Pedro José
Bom dia! Não deu para compreender. Para cada terno (k,j,w) terá apenas uma raiz em x ou nenhuma. Mas para todo natural existe pelo menos um terno que atenda a sua proposição. w=x ; k=1 e j=2. Saudações, PJMS Em 27 de março de 2018 22:28, Israel Meireles Chrisostomo <

Re: [obm-l] Teoria dos numeros

2018-03-27 Por tôpico Israel Meireles Chrisostomo
Está muito geral essas condições, achei que pudesse conseguir alguma restrição a fim de resolver um outro problema, mas talvez esse caminho não é muito apropriado Em 27 de março de 2018 22:10, Claudio Buffara escreveu: > O problema é só esse mesmo? > Não tem nenhum

Re: [obm-l] Teoria dos numeros

2018-03-27 Por tôpico Israel Meireles Chrisostomo
O máximo que eu consigo é considerar uma solução que seja um número primo Em 27 de março de 2018 22:27, Israel Meireles Chrisostomo < israelmchrisost...@gmail.com> escreveu: > Está muito geral essas condições, achei que pudesse conseguir alguma > restrição a fim de resolver um outro problema,

Re: [obm-l] Teoria dos numeros

2018-03-27 Por tôpico Claudio Buffara
O problema é só esse mesmo? Não tem nenhum contexto? Não é dada nenhuma relação entre k, j e w? 2018-03-27 21:27 GMT-03:00 Anderson Torres : > Em 27 de março de 2018 21:06, Israel Meireles Chrisostomo > escreveu: > > Ola pessoal eu

Re: [obm-l] Teoria dos numeros

2018-03-27 Por tôpico Anderson Torres
Em 27 de março de 2018 21:06, Israel Meireles Chrisostomo escreveu: > Ola pessoal eu gostaria de saber quantas são e quais são as soluções > naturais de (x+w)k=xj na variável x, onde k e j e w são naturais dados > (x+w)k=xj xk+wk=xj wk=xj-xk wk=x(j-k) x=wk/(j-k)

[obm-l] Teoria dos numeros

2018-03-27 Por tôpico Israel Meireles Chrisostomo
Ola pessoal eu gostaria de saber quantas são e quais são as soluções naturais de (x+w)k=xj na variável x, onde k e j e w são naturais dados -- Israel Meireles Chrisostomo -- Esta mensagem foi verificada pelo sistema de antiv�rus e acredita-se estar livre de perigo.

Re: [obm-l] Teoria dos numeros

2016-12-22 Por tôpico Pedro José
Bom dia! x= 0 y= 1 e z= 1 ; a = -1, b=-1 e c=-1 -1.0 + -1.1 + -1.1 = -1 + 0 -1 (V) atende a 1 + 1 =1 > = 0 +1 +1 (V) atende b. -1 não é soma de três quadrados de inteiros. Tem que ter mais restrições. Saudações, PJMS Em 20 de dezembro de 2016 19:08, Gabriel Tostes

[obm-l] Teoria dos numeros

2016-12-20 Por tôpico Gabriel Tostes
A,b,c,X,y,z inteiros tais que a) ax^2+by^2+cz^2=abc +2xyz - 1 B) ab+bc+ca>=x^2+y^2+z^2 Provar que a,b,c são somas de 3 quadrados de inteiros -- Esta mensagem foi verificada pelo sistema de antiv�rus e acredita-se estar livre de perigo.

Re: [obm-l] Teoria dos numeros

2013-11-19 Por tôpico terence thirteen
:* sergio marinho smarinh...@yahoo.com.br *To:* obm-l@mat.puc-rio.br *Sent:* Monday, October 28, 2013 4:54 PM *Subject:* Re: [obm-l] Teoria dos numeros Vc poderia me indicar excelentes livros de Teoria dos números e Análise combinatória? Grato. Sérgio Soares. Em Sábado, 3 de Agosto de 2013

Re: [obm-l] Teoria dos numeros

2013-10-28 Por tôpico sergio marinho
Vc poderia me indicar excelentes livros de Teoria dos números e Análise combinatória? Grato.  Sérgio Soares. Em Sábado, 3 de Agosto de 2013 16:47, marcone augusto araújo borges marconeborge...@hotmail.com escreveu: Seja n uma soma de dois numeros triangulares (a^2 + a)/2 e (b^2 +

Re: [obm-l] Teoria dos numeros

2013-10-28 Por tôpico Hermann
Veja na livraria da SBM tem uns muito bons - Original Message - From: sergio marinho To: obm-l@mat.puc-rio.br Sent: Monday, October 28, 2013 4:54 PM Subject: Re: [obm-l] Teoria dos numeros Vc poderia me indicar excelentes livros de Teoria dos números e Análise

[obm-l] Teoria dos numeros

2013-08-03 Por tôpico marcone augusto araújo borges
Seja n uma soma de dois numeros triangulares (a^2 + a)/2 e (b^2 + b)/2.

Re: [obm-l] Teoria dos numeros

2013-08-03 Por tôpico Nehab
Oi, Marcone. Ora, você quer que a soma de dois quadrados dê 2a^2 + 2b^2 + 2a + 2b + 1. O a^2 e o b^2 saem de coisas do tipo (a + b +...)^2 e (a - b +...)^2. Para se livrar do 2ab que aparece nessa coisas, você precisa de um +2ab e de um -2ab... Dai, botando os neurônios para esquentar um

[obm-l] teoria dos numeros

2013-05-10 Por tôpico valdir soares
Ola pessoal, Gostaria de saber, como fazer o problema abaixo : Determine n entre 100 e 1000 , tal que ( 2+ 2^n)/n eh tambem inteiro . Obrigado

Re: [obm-l] teoria dos numeros

2013-05-10 Por tôpico terence thirteen
Aprenda um pouco de inglês: http://ohkawa.cc.it-hiroshima.ac.jp/www.kalva.demon.co.uk/apmo/asoln/asol972.html Em 10 de maio de 2013 06:48, valdir soares valdir.soa...@oi.com.brescreveu: Ola pessoal, Gostaria de saber, como fazer o problema abaixo : Determine n entre 100 e 1000 , tal que

Re: [obm-l] teoria dos numeros

2013-05-10 Por tôpico Bernardo Freitas Paulo da Costa
2013/5/10 terence thirteen peterdirich...@gmail.com: Aprenda um pouco de inglês: http://ohkawa.cc.it-hiroshima.ac.jp/www.kalva.demon.co.uk/apmo/asoln/asol972.html Em 10 de maio de 2013 06:48, valdir soares valdir.soa...@oi.com.br escreveu: Ola pessoal, Gostaria de saber, como fazer o

[obm-l] Teoria dos numeros!

2007-10-03 Por tôpico jose silva
Se possivel, gostaria de contar com a ajuda de voces, para resolver este problema. Mostre que se g e uma raiz impar primitiva de p^m (pelevado a m) com p maior que 2, entao g e uma raiz primitiva de 2p^m (2p elevado a m). Desde ja, muito obrigado! jccardosos.

[obm-l] Teoria dos Numeros

2007-10-01 Por tôpico Marcelo Salhab Brogliato
Olá a todos, estou meio sumido, mas acho q ainda sim posso mandar uma questaozinha q nao consegui resolver.. alias, nem sei c tem solucao... Determine Sum{i=1 ... n} ( k mod i ) apenas para relatar a origem.. eh um problema de programacao, onde 1 = k, n = 10^9... como nao encontrei uma solucao

RE: RES: [obm-l] Teoria dos numeros

2007-06-14 Por tôpico Rhilbert Rivera
= 24 . 72 . 13 5094 = 2 . 32. 283 1698 = 2. 3. 283 Podemos ainda escrever os valores dessas potências como: 10192 = 6. 6. 283 + 4 =1698 . 6 + 4 5094 = 3. 6. 283 = 1698 . 3 1698 = 6 . 283 = 1698 . 1 Logo, o valor de é n = 1698 [[ ]]'s Subject: RES: [obm-l] Teoria dos numeros Date: Tue

Re: [obm-l] Teoria dos numeros

2007-06-12 Por tôpico Paulo Santa Rita
Ola Carissimo Artur e demais colegas desta lista ... OBM-L, E facil ver que 7^4 10200 7^5. Assim, basta considerar ate 7^4. De 7 ate 10199 temos 10199 = 7 + (A-1)*7 = A = 1457 multiplos de 7. Considerando os multiplos de 49 teriamos 10.192 = 49 + (B-1)*49 = B=208 multiplos de 49 e com o

RE: RES: [obm-l] Teoria dos numeros

2007-06-12 Por tôpico Qwert Smith
pelo menos de 3 e o expoente de 3 aumentou de pelo menos 2. From: Artur Costa Steiner [EMAIL PROTECTED] Reply-To: obm-l@mat.puc-rio.br To: obm-l@mat.puc-rio.br Subject: RES: [obm-l] Teoria dos numeros Date: Tue, 12 Jun 2007 13:20:44 -0300 Obrigado Paulo Abraços Artur -Mensagem original

[obm-l] Teoria dos numeros

2007-06-11 Por tôpico Artur Costa Steiner
Estou tentando achar uma solucoa para o seguinte, mas ainda nao consegui: Encontrar o mair valor do ineiro n=0 tal que (10200!)/(504^n) seja inteiro. Nos temos que 504 = 2^3 * 3^2 * 7, assim, o quociente sera inteiro enquanto 10200! contiver os primos 2, 3 e 7 com expoentes no maximo de 3n

Re:[obm-l] Teoria dos numeros?

2006-08-02 Por tôpico claudio\.buffara
De: [EMAIL PROTECTED] Para: obm-l@mat.puc-rio.br Cópia: Data: Tue, 01 Aug 2006 14:37:56 -0400 Assunto: [obm-l] Teoria dos numeros? Liste todos os pares (m,n) para os quais 2^m + 3^n e um quadrado perfeito. Estou supondo que m e n são inteiros não-negativos. Por inspeção

[obm-l] Teoria dos numeros?

2006-08-01 Por tôpico Qwert Smith
Liste todos os pares (m,n) para os quais 2^m + 3^n e um quadrado perfeito. = Instruções para entrar na lista, sair da lista e usar a lista em http://www.mat.puc-rio.br/~nicolau/olimp/obm-l.html

Re: [obm-l] Teoria dos numeros?

2006-08-01 Por tôpico Marcelo Salhab Brogliato
[EMAIL PROTECTED] To: obm-l@mat.puc-rio.br Sent: Tuesday, August 01, 2006 3:37 PM Subject: [obm-l] Teoria dos numeros? Liste todos os pares (m,n) para os quais 2^m + 3^n e um quadrado perfeito. = Instruções para entrar na lista

Re: [obm-l] Teoria dos numeros?

2006-08-01 Por tôpico Manuel Garcia
algoabraços,Salhab- Original Message -From: Qwert Smith [EMAIL PROTECTED]To: obm-l@mat.puc-rio.brSent: Tuesday, August 01, 2006 3:37 PMSubject: [obm-l] Teoria dos numeros? Liste todos os pares (m,n) para os quais 2^m + 3^n e um quadrado perfeito

Re: [obm-l] Teoria dos Numeros[off - topic]

2006-01-28 Por tôpico Danilo Nascimento
Ola Henrique,(x+1)^3-x^3=y^2 -- desenvolva o cubo perfeito. 3x^2+6x+1=y^2 --- multiplique tudo por 4 12x^2+24x+4 = 4y^2--- faça o 4=3+1 12x^2+24x+3=4y^2-1 3(4x^2+8x+1)=(2y-1)(2y+1) 2(2x+1)^2=(2y-1)(2y+1) Dai use que (2y-1)(2y+1) sao primos entre si. Veja q letra b) nao pode ocorrer

Re: [obm-l] Teoria dos Numeros[off - topic]

2006-01-28 Por tôpico Henrique Rennó
Olá Danilo!!! Agradeço a resposta. Acho que tem umas correções no desenvolvimento da expressão a serem feitas. Klaus, Os polígonos são de 4, 6 e 10 lados e não 3, 4 e 6. (x+1)^3-x^3=y^2 -- desenvolva o cubo perfeito. 3x^2+6x+1=y^2 --- multiplique tudo por 4 3x^2 + 3x + 1 = y^2 -- não 6x

Re:[obm-l] Teoria dos Numeros II

2006-01-28 Por tôpico Luiz H\. Barbosa
2) Para quais inteiros n, 18(n^2+3) é cubo perfeito? = Vou resolver esse sem nenhuma ideia esperta: Se 18(n^2+3) é cubo perfeito , então: 18(n^2+3) = x^3 e x0 3.3.2(n^2+3) = x.x^2 Como x é inteiro , temos varios casos: x=2,x=3,x=6, x=9 e x=18 e depois

Re:[obm-l] Teoria dos Numeros II

2006-01-28 Por tôpico Danilo Nascimento
ou veja que 18(n^2+3)=(n+3)^3-(n-3)^3 logo pelo ultimo teorema de fermat, x^n=y^n+z^n, em particular para n=3 a equacao nao possui solucao. dessa forma n+3=0 ou n-3=0 logo n= -+3."Luiz H. Barbosa" [EMAIL PROTECTED] escreveu: 2) Para quais inteiros n, 18(n^2+3) é cubo perfeito? =

Re: [obm-l] Teoria dos Numeros

2006-01-26 Por tôpico Carlos Victor
Olá Klauss , (x+1)^3 - x^3 = y^2 , onde 3(2x+1)^2 = (2y-1)(2y+1) . Observe que podemos concluir que : a) Ou 2y-1 = a^2 e 2y+1 = 3b^2 b) Ou 2y-1 = 3c^2 e 2y+1 = d^2 . Observe que 3b^2 = a^2 +2 é a única que pode ocorrer e, como a é ímpar , podemos escrever a = 2t +1 e 4y = 2(a^2+1) implicando

Re: [obm-l] Teoria dos Numeros[off - topic]

2006-01-26 Por tôpico Klaus Ferraz
Vlw. Onde consigo esse livro, POWER PLAY de EDWARD J. BARBEAU da MAA Carlos Victor [EMAIL PROTECTED] escreveu: Olá Klauss ,(x+1)^3 - x^3 = y^2 , onde 3(2x+1)^2 = (2y-1)(2y+1) . Observe que podemos concluir que :a) Ou 2y-1 = a^2 e 2y+1 = 3b^2 b) Ou 2y-1 = 3c^2 e 2y+1 = d^2 .Observe que 3b^2 = a^2

Re: [obm-l] Teoria dos Numeros[off - topic]

2006-01-26 Por tôpico Igor Castro
na www.amazon.com - Original Message - From: Klaus Ferraz To: obm-l@mat.puc-rio.br Sent: Thursday, January 26, 2006 7:00 PM Subject: Re: [obm-l] Teoria dos Numeros[off - topic] Vlw. Onde consigo esse livro, POWER PLAY de EDWARD J. BARBEAU da MAA

[obm-l] Teoria dos Numeros

2006-01-24 Por tôpico Klaus Ferraz
Mostre que a diferença entreos cubos de doisnumeros inteiros consecutivos é igual ao quadrado de um inteiro, entao esse inteiro é igual a soma dos quadrados de dois inteiros consecutivos. Ex: 8^3-7^3=169. 2^2+3^2=13.Grato. Yahoo! doce lar. Faça do Yahoo! sua homepage.

[obm-l] teoria dos numeros!!!

2005-12-29 Por tôpico diego andres
alguem me mostra um jeito fácil de descobrir um sistema completo de residuos modulo 7onde todos os elementos sao primos... valeu ai pessoal,Diego Yahoo! doce lar. Faça do Yahoo! sua homepage.

[obm-l] teoria dos numeros

2005-10-19 Por tôpico Rodrigo Augusto
boa tarde a todos, quem me ajuda com esse? sabemos que existem infinitos numeros primos da forma 4k + 3. dado um inteiro b e sendo S o conjunto de todos os primos da forma p = 4k + 3 , onde p não divide b. a questão é: existem dois primos (4k + 3) e (4q + 3) em S de tal forma que (k + 1) e (q

Re:[obm-l] teoria dos numeros

2005-10-19 Por tôpico claudio\.buffara
Que tal 7 e 19? 7 = 4*1 + 3 e 19 = 4*4 + 3 mdc(1+1,4+1) = 1. []s, Claudio. De: [EMAIL PROTECTED] Para: obm-l@mat.puc-rio.br Cópia: Data: Wed, 19 Oct 2005 16:01:03 -0200 Assunto: [obm-l] teoria dos numeros boa tarde a todos, quem me ajuda com esse? sabemos que existem

[obm-l] Teoria dos numeros

2005-02-06 Por tôpico kleinad
Aqui vai um probleminha (que eu achei!) legal: Seja p um número primo. Seja A_d = { a em (Z/pZ)* tal que ord(a) = d } para cada d divisor de fi(p), onde (Z/pZ)* = (Z/pZ) - { 0 } e fi é a função de Euler. Definimos f(d) = soma de todos os elementos de A_d. Prove que f(d) == mi(d) (mod p) para todo

[obm-l] Teoria dos Numeros-Soluçao de um Hojoo Lee

2004-05-03 Por tôpico Johann Peter Gustav Lejeune Dirichlet
Ola turma!!! Como disse o Claudio, vamos nos esbaldar em problemas.TN nao e meu preferido mas... Acabei de dar uma passada pelo site do Hojoo Lee e fiz esse problema da apostila de TN.Vejam so que legal... "Seja p um primo impar. Prove que existem infinitos primos x tais que 2p divide x-1".

[obm-l] Re: [obm-l] Teoria dos Numeros-Soluçao de um Hojoo Lee

2004-05-03 Por tôpico Domingos Jr.
Seja p um primo impar. Prove que existem infinitos primos x tais que 2p divide x-1. considere a PA {(2p)n + 1 : n pertence a Z} como mdc(2p, 1) = 1 temos, pelo seu teorema (Dirichlet) que tal PA possui infinitos primos. ou seja, este problema é um caso particular do super-canhão-teorema de PAs.

Re: [obm-l] Teoria dos Numeros

2004-04-20 Por tôpico Claudio Buffara
Title: Re: [obm-l] Teoria dos Numeros on 17.04.04 10:56, Johann Peter Gustav Lejeune Dirichlet at [EMAIL PROTECTED] wrote: Seja X o conjunto dos primos tais que se a e b sao dois elementos dele entao ab+4 e a^2+4 tambem estao.Prove ou disprove: X e vazio Alem da solucao do Gugu, existe uma

Re: [obm-l] Teoria dos Numeros

2004-04-20 Por tôpico Johann Peter Gustav Lejeune Dirichlet
Pode-se dizer que sim.Eu preferi escrever desse jeito para nao dar margem a duvidas --- Carlos Gustavo Tamm de Araujo Moreira [EMAIL PROTECTED] escreveu: Caros Claudio e Dirichlet, Bacana esse problema. Vamos la': Dadas essas condicoes, se a pertence a X entao

Re: [obm-l] Teoria dos Numeros

2004-04-20 Por tôpico Johann Peter Gustav Lejeune Dirichlet
Uma resposta para o Claudio: Este problema eu propus logo quando eu entrei na lista.Ninguem tinha mandado nada sobre isso.resolvi mandar de novo agora que vi em uns papeis de matematica olimpica que eu guardava. Eu ate um tempinho atras so tinha limitado os caras em algumas congruencias,mas nada

Re: [obm-l] Teoria dos Numeros

2004-04-20 Por tôpico Johann Peter Gustav Lejeune Dirichlet
Onde esta ela? Alias sera que da para generalizar esse quatro? --- Claudio Buffara [EMAIL PROTECTED] escreveu: on 17.04.04 10:56, Johann Peter Gustav Lejeune Dirichlet at [EMAIL PROTECTED] wrote: Seja X o conjunto dos primos tais que se a e b sao dois elementos dele entao ab+4 e a^2+4

Re: [obm-l] Teoria dos Numeros

2004-04-20 Por tôpico Johann Peter Gustav Lejeune Dirichlet
A soluçao do Gugu, como ja era de se esperar, foi demais!! --- Johann Peter Gustav Lejeune Dirichlet [EMAIL PROTECTED] escreveu: Uma resposta para o Claudio: Este problema eu propus logo quando eu entrei na lista.Ninguem tinha mandado nada sobre isso.resolvi mandar de novo agora que vi em

Re: [obm-l] Teoria dos Numeros

2004-04-20 Por tôpico Claudio Buffara
Analise os 7 casos possiveis, a == 0, 1, 2, 3, 4, 5 e 6 (mod 7). Alias, nem precisa analisar todos, jah que k^2 == (7-k)^2 (mod 7). A solucao sai facilmente. Sobre a generalizacao, suponhamos que a condicao seja: a, b pertencem a X == ab + k pertence a X, com k = inteiro fixo. Entao: a

Re: [obm-l] Teoria dos Numeros

2004-04-20 Por tôpico Johann Peter Gustav Lejeune Dirichlet
Qualquer coincidencia e mera semelhança...Claudio Buffara [EMAIL PROTECTED] wrote: Analise os 7 casos possiveis, a == 0, 1, 2, 3, 4, 5 e 6 (mod 7). Alias, nemprecisa analisar todos, jah que k^2 == (7-k)^2 (mod 7). A solucao saifacilmente.Sobre a generalizacao, suponhamos que a condicao seja:a,

Re: [obm-l] Teoria dos Numeros

2004-04-19 Por tôpico Ricardo Bittencourt
Johann Peter Gustav Lejeune Dirichlet wrote: (a,b = ab+4 e a^2+4) Mas espere, 6m-1=m-1 mod 5.Logo (6m-1)(6n-1)+4=mn-m-n mod 5.Sera que da para arrancar alguem mod 5? Se mn=m+n mod 5 entao nao da primo Eu já consegui mostrar que todos os elementos do conjunto são da forma 30k+23, mas ainda

Re: [obm-l] Teoria dos Numeros

2004-04-19 Por tôpico Claudio Buffara
on 19.04.04 12:54, Ricardo Bittencourt at [EMAIL PROTECTED] wrote: Johann Peter Gustav Lejeune Dirichlet wrote: (a,b = ab+4 e a^2+4) Mas espere, 6m-1=m-1 mod 5.Logo (6m-1)(6n-1)+4=mn-m-n mod 5.Sera que da para arrancar alguem mod 5? Se mn=m+n mod 5 entao nao da primo Eu já consegui

Re: [obm-l] Teoria dos Numeros

2004-04-19 Por tôpico Carlos Gustavo Tamm de Araujo Moreira
Caros Claudio e Dirichlet, Bacana esse problema. Vamos la': Dadas essas condicoes, se a pertence a X entao b(n)=a^(n+2)+4.(a^n+a^(n-1)+...+a+1)=a^(n+2)+4.(a^(n+1)-1)/(a-1) pertence a X para todo n, mas para todo primo q (digamos q=b(0)=a^2+4), b(n) (mod q) e' periodica com periodo

[obm-l] Teoria dos Numeros

2004-04-17 Por tôpico Johann Peter Gustav Lejeune Dirichlet
Seja X o conjunto dos primos tais que se a e b sao dois elementos dele entao ab+4 e a^2+4 tambem estao.Prove ou disprove: X e vazio TRANSIRE SVVM PECTVS MVNDOQVE POTIRI CONGREGATI EX TOTO ORBE MATHEMATICI OB SCRIPTA INSIGNIA TRIBVERE Fields Medal(John Charles Fields)Yahoo! Messenger - Fale com

Re: [obm-l] Teoria dos Numeros

2004-04-17 Por tôpico Claudio Buffara
Title: Re: [obm-l] Teoria dos Numeros on 17.04.04 10:56, Johann Peter Gustav Lejeune Dirichlet at [EMAIL PROTECTED] wrote: Seja X o conjunto dos primos tais que se a e b sao dois elementos dele entao ab+4 e a^2+4 tambem estao.Prove ou disprove: X e vazio Inicio de solucao: Suponhamos que X

[obm-l] teoria dos numeros

2004-01-21 Por tôpico levi queiroz
Pessoal da lista , eu estou enviando para de vocês quatro proposições minhas que eu mesmo demonstrei e no entanto eu não sei se constam dentro da Teoria dos Números. Gostaria da ajuda de vocês. Proposição 1: Se p 3e p+2 são primos gêmeos então p +1 = 6k, para algum k inteiro Como pé primo ímpar

Re: [obm-l] teoria dos numeros

2004-01-21 Por tôpico Nicolau C. Saldanha
On Wed, Jan 21, 2004 at 04:46:28PM -0300, levi queiroz wrote: Pessoal da lista , eu estou enviando para de vocês quatro proposições minhas que eu mesmo demonstrei e no entanto eu não sei se constam dentro da Teoria dos Números. Gostaria da ajuda de vocês. Proposição 1: Se p 3 e p+2 são

[obm-l] teoria dos numeros - parte II

2004-01-21 Por tôpico levi queiroz
Obrigado Professor Nicolau Saldanha! Eu queria saber se os resultados eram conhecidos. Eu cheguei a estes resultados sem saber que eles já eram de domínio público. De qualquer maneira para mim foi um grande prazer ter encontrado estes resultados. Foi procurar nas fontes indicadas pelo senhor para

Re: [obm-l] Teoria dos numeros

2003-09-15 Por tôpico Marcelo Souza
-l] Teoria dos numeros Date: Sun, 14 Sep 2003 20:37:26 -0300 Prove as seguintes afirmações: a) Se a é um inteiro ímpar, então 24 divide a*(a^2 - 1) b) Se a e b são inteiros impares, entao 8 divide a^2 - b^2 No caso do item b) pensei em considerar a = 4k-1 e b = 4k+1. Eu perco a generalidade se fizer

[obm-l] Teoria dos numeros

2003-09-14 Por tôpico Henrique Patrício Sant'Anna Branco
Prove as seguintes afirmações: a) Se a é um inteiro ímpar, então 24 divide a*(a^2 - 1) b) Se a e b são inteiros impares, entao 8 divide a^2 - b^2 No caso do item b) pensei em considerar a = 4k-1 e b = 4k+1. Eu perco a generalidade se fizer algo assim? Grato, Henrique.

Re: [obm-l] Teoria dos numeros

2003-09-14 Por tôpico Murilo
*2*2 = 8 - Original Message - From: Henrique Patrício Sant'Anna Branco [EMAIL PROTECTED] To: OBM [EMAIL PROTECTED] Sent: Sunday, September 14, 2003 8:37 PM Subject: [obm-l] Teoria dos numeros Prove as seguintes afirmações: a) Se a é um inteiro ímpar, então 24 divide a*(a^2 - 1) b) Se

Re: [obm-l] Teoria dos numeros

2003-09-14 Por tôpico Claudio Buffara
on 14.09.03 20:37, Henrique Patrício Sant'Anna Branco at [EMAIL PROTECTED] wrote: Prove as seguintes afirmações: a) Se a é um inteiro ímpar, então 24 divide a*(a^2 - 1) b) Se a e b são inteiros impares, entao 8 divide a^2 - b^2 No caso do item b) pensei em considerar a = 4k-1 e b = 4k+1. Eu

Re: [obm-l] Teoria dos Numeros

2002-07-15 Por tôpico Nicolau C. Saldanha
On Sun, Jul 14, 2002 at 04:45:19PM -0300, adr.scr.m wrote: Alguem poderia fazer essas questoes para mim ? Determine todos os primos que sao a soma e a diferenca de 2 primos. 5 = 3 + 2 = 7 - 2 'e o 'unico. Basta observar que 'e indispens'avel usar o primo 2 e que o 'unico caso em que

Re: [obm-l] Teoria dos Numeros

2002-07-15 Por tôpico Nicolau C. Saldanha
On Sun, Jul 14, 2002 at 04:45:19PM -0300, adr.scr.m wrote: Determine todos inteiros positivos x,y,z,tais que z divide xy-1,x divide zy-1 e y divide zx-1. Este problema 'e bem legal. Vou pular umas linhas antes de dar a solu,c~ao para que os outros tentem fazer sozinhos, vale a pena.

[obm-l] Teoria dos Numeros

2002-07-14 Por tôpico adr.scr.m
Alguem poderia fazer essas questoes para mim ? Determine todos os primos que sao a soma e a diferenca de 2 primos. Determine todos inteiros positivos x,y,z,tais que z divide xy-1,x divide zy-1 e y divide zx-1. Obrigado. Adriano.

[obm-l] Teoria dos Numeros

2002-03-06 Por tôpico osvaldo correa do nascimento junior
Olá lista, Desculpe-me o assunto off-topic mas, alguem da lista teria o livro de Teoria Elementar dos Numeros de edgard de Alencar Filho. Valeu !!! Osvaldo Correa = Instruções para entrar na lista, sair da lista e usar a